Prove $x equiv a pmod{p}$ and $x equiv a pmod{q}$ then $x equiv apmod{pq}$











up vote
3
down vote

favorite












$p$ and $q$ are distinct primes.



Where can I start with this proof?
It looks similar to the Chinese Remainder Theorem, but that deals with two different a values.










share|cite|improve this question
























  • but that deals with two different $a$ values - What makes you think that?
    – anon
    Sep 3 '12 at 15:59










  • Because for the explanations of Chinese Remainder Theorem I've read, they use something like a = x (mod p) and a = y (mod p)
    – Takkun
    Sep 3 '12 at 16:03






  • 2




    A couple of variables denoted with two different letters may take on two different values, but not necessarily unless they are explicitly stated to have distinct values.
    – anon
    Sep 3 '12 at 16:04

















up vote
3
down vote

favorite












$p$ and $q$ are distinct primes.



Where can I start with this proof?
It looks similar to the Chinese Remainder Theorem, but that deals with two different a values.










share|cite|improve this question
























  • but that deals with two different $a$ values - What makes you think that?
    – anon
    Sep 3 '12 at 15:59










  • Because for the explanations of Chinese Remainder Theorem I've read, they use something like a = x (mod p) and a = y (mod p)
    – Takkun
    Sep 3 '12 at 16:03






  • 2




    A couple of variables denoted with two different letters may take on two different values, but not necessarily unless they are explicitly stated to have distinct values.
    – anon
    Sep 3 '12 at 16:04















up vote
3
down vote

favorite









up vote
3
down vote

favorite











$p$ and $q$ are distinct primes.



Where can I start with this proof?
It looks similar to the Chinese Remainder Theorem, but that deals with two different a values.










share|cite|improve this question















$p$ and $q$ are distinct primes.



Where can I start with this proof?
It looks similar to the Chinese Remainder Theorem, but that deals with two different a values.







number-theory






share|cite|improve this question















share|cite|improve this question













share|cite|improve this question




share|cite|improve this question








edited Sep 3 '12 at 16:08









Thomas Russell

7,80632550




7,80632550










asked Sep 3 '12 at 15:57









Takkun

263211




263211












  • but that deals with two different $a$ values - What makes you think that?
    – anon
    Sep 3 '12 at 15:59










  • Because for the explanations of Chinese Remainder Theorem I've read, they use something like a = x (mod p) and a = y (mod p)
    – Takkun
    Sep 3 '12 at 16:03






  • 2




    A couple of variables denoted with two different letters may take on two different values, but not necessarily unless they are explicitly stated to have distinct values.
    – anon
    Sep 3 '12 at 16:04




















  • but that deals with two different $a$ values - What makes you think that?
    – anon
    Sep 3 '12 at 15:59










  • Because for the explanations of Chinese Remainder Theorem I've read, they use something like a = x (mod p) and a = y (mod p)
    – Takkun
    Sep 3 '12 at 16:03






  • 2




    A couple of variables denoted with two different letters may take on two different values, but not necessarily unless they are explicitly stated to have distinct values.
    – anon
    Sep 3 '12 at 16:04


















but that deals with two different $a$ values - What makes you think that?
– anon
Sep 3 '12 at 15:59




but that deals with two different $a$ values - What makes you think that?
– anon
Sep 3 '12 at 15:59












Because for the explanations of Chinese Remainder Theorem I've read, they use something like a = x (mod p) and a = y (mod p)
– Takkun
Sep 3 '12 at 16:03




Because for the explanations of Chinese Remainder Theorem I've read, they use something like a = x (mod p) and a = y (mod p)
– Takkun
Sep 3 '12 at 16:03




2




2




A couple of variables denoted with two different letters may take on two different values, but not necessarily unless they are explicitly stated to have distinct values.
– anon
Sep 3 '12 at 16:04






A couple of variables denoted with two different letters may take on two different values, but not necessarily unless they are explicitly stated to have distinct values.
– anon
Sep 3 '12 at 16:04












3 Answers
3






active

oldest

votes

















up vote
6
down vote



accepted










Hint $ $ Below are few proofs of this constant-case CCRT of Chinese Remainder Theorem (CRT). The first three proofs work for for arbitrary coprime naturals $rm,p,q.$



$rm(1) x equiv apmod {pq}:$ is clearly a solution, and the solution is $color{#C00}{unique}$ mod $rm,pq,$ by CRT.



$rm(2) p,q:|:x!-!aiff lcm(p,q):|:x!-!a.:$ Further $rm:(p,q)=1iff:lcm(p,q) = pq.$



$(3) , $ By Euclid's Lemma: $rm:(p,q)=1, p:|:qn =:x!-!a:Rightarrow:p:|:n:Rightarrow:pq:|:nq = x!-!a.$



$rm(4) , $ Since the prime factorization of $rm,x!-!a,$ is $color{#C00}{unique}$, and the prime $rm,p,$ occurs in one factorization, and the distinct prime $rm,q,$ occurs in another, both factorizations are the same up to order, hence contain both $rm,p,$ and $rm,q,:$ hence have $rm,pq,$ as a divisor.



Remark $ $ This constant-case optimization of CRT arises frequently in practice so is well-worth memorizing, e.g. see some prior posts for many examples.



Quite frequently, $color{#C00}{uniqueness} theorems$ provide powerful tools for proving equalities.






share|cite|improve this answer























  • in (2) do you mean to say $gcd(p, q) = 1 iff lcm(p, q) = pq$
    – CodeKingPlusPlus
    Sep 23 '12 at 23:17










  • @Code Yes, $rm:(x,y):$ means $rm:gcd(x,y):$ in number theory (common notation).
    – Bill Dubuque
    Sep 23 '12 at 23:37


















up vote
4
down vote













Let $[A,B]=lcm(A,B)$ and $(A,B)=gcd(A,B)$



If $p,q$ are different integers, $pmid(x-a)$ and $qmid(x-a)implies [p,q]mid(x-a)$



We know $[p,q]cdot (p,q)=pcdot q$



If $(p,q)=1, [p,q]=pcdot q$



If $p,q$ are distinct primes, $(p,q)=1$






share|cite|improve this answer






























    up vote
    3
    down vote













    Let $y=x-a$. We want to show that if $p$ divides $y$ and $q$ divides $y$ then $pq$ divides $y$.



    Since $p$ divides $y$, we have $y=pz$ for some $z$. Thus $q$ divides $pz$. Since $q$ is prime, this implies $q$ divides $p$ or $q$ divides $z$. But $q$ cannot divide $p$, so $q$ divides $z$. Suppose that $z=qw$. Then $y=pqw$.






    share|cite|improve this answer























      Your Answer





      StackExchange.ifUsing("editor", function () {
      return StackExchange.using("mathjaxEditing", function () {
      StackExchange.MarkdownEditor.creationCallbacks.add(function (editor, postfix) {
      StackExchange.mathjaxEditing.prepareWmdForMathJax(editor, postfix, [["$", "$"], ["\\(","\\)"]]);
      });
      });
      }, "mathjax-editing");

      StackExchange.ready(function() {
      var channelOptions = {
      tags: "".split(" "),
      id: "69"
      };
      initTagRenderer("".split(" "), "".split(" "), channelOptions);

      StackExchange.using("externalEditor", function() {
      // Have to fire editor after snippets, if snippets enabled
      if (StackExchange.settings.snippets.snippetsEnabled) {
      StackExchange.using("snippets", function() {
      createEditor();
      });
      }
      else {
      createEditor();
      }
      });

      function createEditor() {
      StackExchange.prepareEditor({
      heartbeatType: 'answer',
      convertImagesToLinks: true,
      noModals: true,
      showLowRepImageUploadWarning: true,
      reputationToPostImages: 10,
      bindNavPrevention: true,
      postfix: "",
      imageUploader: {
      brandingHtml: "Powered by u003ca class="icon-imgur-white" href="https://imgur.com/"u003eu003c/au003e",
      contentPolicyHtml: "User contributions licensed under u003ca href="https://creativecommons.org/licenses/by-sa/3.0/"u003ecc by-sa 3.0 with attribution requiredu003c/au003e u003ca href="https://stackoverflow.com/legal/content-policy"u003e(content policy)u003c/au003e",
      allowUrls: true
      },
      noCode: true, onDemand: true,
      discardSelector: ".discard-answer"
      ,immediatelyShowMarkdownHelp:true
      });


      }
      });














       

      draft saved


      draft discarded


















      StackExchange.ready(
      function () {
      StackExchange.openid.initPostLogin('.new-post-login', 'https%3a%2f%2fmath.stackexchange.com%2fquestions%2f190514%2fprove-x-equiv-a-pmodp-and-x-equiv-a-pmodq-then-x-equiv-a-pmodpq%23new-answer', 'question_page');
      }
      );

      Post as a guest















      Required, but never shown

























      3 Answers
      3






      active

      oldest

      votes








      3 Answers
      3






      active

      oldest

      votes









      active

      oldest

      votes






      active

      oldest

      votes








      up vote
      6
      down vote



      accepted










      Hint $ $ Below are few proofs of this constant-case CCRT of Chinese Remainder Theorem (CRT). The first three proofs work for for arbitrary coprime naturals $rm,p,q.$



      $rm(1) x equiv apmod {pq}:$ is clearly a solution, and the solution is $color{#C00}{unique}$ mod $rm,pq,$ by CRT.



      $rm(2) p,q:|:x!-!aiff lcm(p,q):|:x!-!a.:$ Further $rm:(p,q)=1iff:lcm(p,q) = pq.$



      $(3) , $ By Euclid's Lemma: $rm:(p,q)=1, p:|:qn =:x!-!a:Rightarrow:p:|:n:Rightarrow:pq:|:nq = x!-!a.$



      $rm(4) , $ Since the prime factorization of $rm,x!-!a,$ is $color{#C00}{unique}$, and the prime $rm,p,$ occurs in one factorization, and the distinct prime $rm,q,$ occurs in another, both factorizations are the same up to order, hence contain both $rm,p,$ and $rm,q,:$ hence have $rm,pq,$ as a divisor.



      Remark $ $ This constant-case optimization of CRT arises frequently in practice so is well-worth memorizing, e.g. see some prior posts for many examples.



      Quite frequently, $color{#C00}{uniqueness} theorems$ provide powerful tools for proving equalities.






      share|cite|improve this answer























      • in (2) do you mean to say $gcd(p, q) = 1 iff lcm(p, q) = pq$
        – CodeKingPlusPlus
        Sep 23 '12 at 23:17










      • @Code Yes, $rm:(x,y):$ means $rm:gcd(x,y):$ in number theory (common notation).
        – Bill Dubuque
        Sep 23 '12 at 23:37















      up vote
      6
      down vote



      accepted










      Hint $ $ Below are few proofs of this constant-case CCRT of Chinese Remainder Theorem (CRT). The first three proofs work for for arbitrary coprime naturals $rm,p,q.$



      $rm(1) x equiv apmod {pq}:$ is clearly a solution, and the solution is $color{#C00}{unique}$ mod $rm,pq,$ by CRT.



      $rm(2) p,q:|:x!-!aiff lcm(p,q):|:x!-!a.:$ Further $rm:(p,q)=1iff:lcm(p,q) = pq.$



      $(3) , $ By Euclid's Lemma: $rm:(p,q)=1, p:|:qn =:x!-!a:Rightarrow:p:|:n:Rightarrow:pq:|:nq = x!-!a.$



      $rm(4) , $ Since the prime factorization of $rm,x!-!a,$ is $color{#C00}{unique}$, and the prime $rm,p,$ occurs in one factorization, and the distinct prime $rm,q,$ occurs in another, both factorizations are the same up to order, hence contain both $rm,p,$ and $rm,q,:$ hence have $rm,pq,$ as a divisor.



      Remark $ $ This constant-case optimization of CRT arises frequently in practice so is well-worth memorizing, e.g. see some prior posts for many examples.



      Quite frequently, $color{#C00}{uniqueness} theorems$ provide powerful tools for proving equalities.






      share|cite|improve this answer























      • in (2) do you mean to say $gcd(p, q) = 1 iff lcm(p, q) = pq$
        – CodeKingPlusPlus
        Sep 23 '12 at 23:17










      • @Code Yes, $rm:(x,y):$ means $rm:gcd(x,y):$ in number theory (common notation).
        – Bill Dubuque
        Sep 23 '12 at 23:37













      up vote
      6
      down vote



      accepted







      up vote
      6
      down vote



      accepted






      Hint $ $ Below are few proofs of this constant-case CCRT of Chinese Remainder Theorem (CRT). The first three proofs work for for arbitrary coprime naturals $rm,p,q.$



      $rm(1) x equiv apmod {pq}:$ is clearly a solution, and the solution is $color{#C00}{unique}$ mod $rm,pq,$ by CRT.



      $rm(2) p,q:|:x!-!aiff lcm(p,q):|:x!-!a.:$ Further $rm:(p,q)=1iff:lcm(p,q) = pq.$



      $(3) , $ By Euclid's Lemma: $rm:(p,q)=1, p:|:qn =:x!-!a:Rightarrow:p:|:n:Rightarrow:pq:|:nq = x!-!a.$



      $rm(4) , $ Since the prime factorization of $rm,x!-!a,$ is $color{#C00}{unique}$, and the prime $rm,p,$ occurs in one factorization, and the distinct prime $rm,q,$ occurs in another, both factorizations are the same up to order, hence contain both $rm,p,$ and $rm,q,:$ hence have $rm,pq,$ as a divisor.



      Remark $ $ This constant-case optimization of CRT arises frequently in practice so is well-worth memorizing, e.g. see some prior posts for many examples.



      Quite frequently, $color{#C00}{uniqueness} theorems$ provide powerful tools for proving equalities.






      share|cite|improve this answer














      Hint $ $ Below are few proofs of this constant-case CCRT of Chinese Remainder Theorem (CRT). The first three proofs work for for arbitrary coprime naturals $rm,p,q.$



      $rm(1) x equiv apmod {pq}:$ is clearly a solution, and the solution is $color{#C00}{unique}$ mod $rm,pq,$ by CRT.



      $rm(2) p,q:|:x!-!aiff lcm(p,q):|:x!-!a.:$ Further $rm:(p,q)=1iff:lcm(p,q) = pq.$



      $(3) , $ By Euclid's Lemma: $rm:(p,q)=1, p:|:qn =:x!-!a:Rightarrow:p:|:n:Rightarrow:pq:|:nq = x!-!a.$



      $rm(4) , $ Since the prime factorization of $rm,x!-!a,$ is $color{#C00}{unique}$, and the prime $rm,p,$ occurs in one factorization, and the distinct prime $rm,q,$ occurs in another, both factorizations are the same up to order, hence contain both $rm,p,$ and $rm,q,:$ hence have $rm,pq,$ as a divisor.



      Remark $ $ This constant-case optimization of CRT arises frequently in practice so is well-worth memorizing, e.g. see some prior posts for many examples.



      Quite frequently, $color{#C00}{uniqueness} theorems$ provide powerful tools for proving equalities.







      share|cite|improve this answer














      share|cite|improve this answer



      share|cite|improve this answer








      edited 2 days ago

























      answered Sep 3 '12 at 16:18









      Bill Dubuque

      206k29189621




      206k29189621












      • in (2) do you mean to say $gcd(p, q) = 1 iff lcm(p, q) = pq$
        – CodeKingPlusPlus
        Sep 23 '12 at 23:17










      • @Code Yes, $rm:(x,y):$ means $rm:gcd(x,y):$ in number theory (common notation).
        – Bill Dubuque
        Sep 23 '12 at 23:37


















      • in (2) do you mean to say $gcd(p, q) = 1 iff lcm(p, q) = pq$
        – CodeKingPlusPlus
        Sep 23 '12 at 23:17










      • @Code Yes, $rm:(x,y):$ means $rm:gcd(x,y):$ in number theory (common notation).
        – Bill Dubuque
        Sep 23 '12 at 23:37
















      in (2) do you mean to say $gcd(p, q) = 1 iff lcm(p, q) = pq$
      – CodeKingPlusPlus
      Sep 23 '12 at 23:17




      in (2) do you mean to say $gcd(p, q) = 1 iff lcm(p, q) = pq$
      – CodeKingPlusPlus
      Sep 23 '12 at 23:17












      @Code Yes, $rm:(x,y):$ means $rm:gcd(x,y):$ in number theory (common notation).
      – Bill Dubuque
      Sep 23 '12 at 23:37




      @Code Yes, $rm:(x,y):$ means $rm:gcd(x,y):$ in number theory (common notation).
      – Bill Dubuque
      Sep 23 '12 at 23:37










      up vote
      4
      down vote













      Let $[A,B]=lcm(A,B)$ and $(A,B)=gcd(A,B)$



      If $p,q$ are different integers, $pmid(x-a)$ and $qmid(x-a)implies [p,q]mid(x-a)$



      We know $[p,q]cdot (p,q)=pcdot q$



      If $(p,q)=1, [p,q]=pcdot q$



      If $p,q$ are distinct primes, $(p,q)=1$






      share|cite|improve this answer



























        up vote
        4
        down vote













        Let $[A,B]=lcm(A,B)$ and $(A,B)=gcd(A,B)$



        If $p,q$ are different integers, $pmid(x-a)$ and $qmid(x-a)implies [p,q]mid(x-a)$



        We know $[p,q]cdot (p,q)=pcdot q$



        If $(p,q)=1, [p,q]=pcdot q$



        If $p,q$ are distinct primes, $(p,q)=1$






        share|cite|improve this answer

























          up vote
          4
          down vote










          up vote
          4
          down vote









          Let $[A,B]=lcm(A,B)$ and $(A,B)=gcd(A,B)$



          If $p,q$ are different integers, $pmid(x-a)$ and $qmid(x-a)implies [p,q]mid(x-a)$



          We know $[p,q]cdot (p,q)=pcdot q$



          If $(p,q)=1, [p,q]=pcdot q$



          If $p,q$ are distinct primes, $(p,q)=1$






          share|cite|improve this answer














          Let $[A,B]=lcm(A,B)$ and $(A,B)=gcd(A,B)$



          If $p,q$ are different integers, $pmid(x-a)$ and $qmid(x-a)implies [p,q]mid(x-a)$



          We know $[p,q]cdot (p,q)=pcdot q$



          If $(p,q)=1, [p,q]=pcdot q$



          If $p,q$ are distinct primes, $(p,q)=1$







          share|cite|improve this answer














          share|cite|improve this answer



          share|cite|improve this answer








          edited Sep 3 '12 at 16:06

























          answered Sep 3 '12 at 16:01









          lab bhattacharjee

          220k15154271




          220k15154271






















              up vote
              3
              down vote













              Let $y=x-a$. We want to show that if $p$ divides $y$ and $q$ divides $y$ then $pq$ divides $y$.



              Since $p$ divides $y$, we have $y=pz$ for some $z$. Thus $q$ divides $pz$. Since $q$ is prime, this implies $q$ divides $p$ or $q$ divides $z$. But $q$ cannot divide $p$, so $q$ divides $z$. Suppose that $z=qw$. Then $y=pqw$.






              share|cite|improve this answer



























                up vote
                3
                down vote













                Let $y=x-a$. We want to show that if $p$ divides $y$ and $q$ divides $y$ then $pq$ divides $y$.



                Since $p$ divides $y$, we have $y=pz$ for some $z$. Thus $q$ divides $pz$. Since $q$ is prime, this implies $q$ divides $p$ or $q$ divides $z$. But $q$ cannot divide $p$, so $q$ divides $z$. Suppose that $z=qw$. Then $y=pqw$.






                share|cite|improve this answer

























                  up vote
                  3
                  down vote










                  up vote
                  3
                  down vote









                  Let $y=x-a$. We want to show that if $p$ divides $y$ and $q$ divides $y$ then $pq$ divides $y$.



                  Since $p$ divides $y$, we have $y=pz$ for some $z$. Thus $q$ divides $pz$. Since $q$ is prime, this implies $q$ divides $p$ or $q$ divides $z$. But $q$ cannot divide $p$, so $q$ divides $z$. Suppose that $z=qw$. Then $y=pqw$.






                  share|cite|improve this answer














                  Let $y=x-a$. We want to show that if $p$ divides $y$ and $q$ divides $y$ then $pq$ divides $y$.



                  Since $p$ divides $y$, we have $y=pz$ for some $z$. Thus $q$ divides $pz$. Since $q$ is prime, this implies $q$ divides $p$ or $q$ divides $z$. But $q$ cannot divide $p$, so $q$ divides $z$. Suppose that $z=qw$. Then $y=pqw$.







                  share|cite|improve this answer














                  share|cite|improve this answer



                  share|cite|improve this answer








                  edited Sep 3 '12 at 16:30

























                  answered Sep 3 '12 at 16:08









                  André Nicolas

                  450k36419801




                  450k36419801






























                       

                      draft saved


                      draft discarded



















































                       


                      draft saved


                      draft discarded














                      StackExchange.ready(
                      function () {
                      StackExchange.openid.initPostLogin('.new-post-login', 'https%3a%2f%2fmath.stackexchange.com%2fquestions%2f190514%2fprove-x-equiv-a-pmodp-and-x-equiv-a-pmodq-then-x-equiv-a-pmodpq%23new-answer', 'question_page');
                      }
                      );

                      Post as a guest















                      Required, but never shown





















































                      Required, but never shown














                      Required, but never shown












                      Required, but never shown







                      Required, but never shown

































                      Required, but never shown














                      Required, but never shown












                      Required, but never shown







                      Required, but never shown







                      Popular posts from this blog

                      Can a sorcerer learn a 5th-level spell early by creating spell slots using the Font of Magic feature?

                      Does disintegrating a polymorphed enemy still kill it after the 2018 errata?

                      A Topological Invariant for $pi_3(U(n))$